You are on page 1of 25

Paper No.

3 Analytical
A new bank has decided to stay open only on weekends all day Saturday and Sunday, and no other days. The bank has hired two mangers (U and V) for tellers (W,X,Y, and Z), and two operations officers (S and T), for a total of exactly eight full-time employees. No part time employees are hired. Each employee works a complete day when working. A manager must be on duty each day. The mangers cannot work on the same day. At least two tellers must be working on the same day. W and X will not work on the same day. S and Z will only work on Saturday. No employee can work on consecutive days, but each employee must work on Saturday or Sunday. Question:

What is the greatest number of employees that can work on Saturday?


A. 2 B. 3 C. 4 D. 5 E. 6

Given Answer: Explanation:

Correct Answer

Another possible display might look like this:

Five employees, U or V, X or W ,Z, S or T are the greatest number to work on Saturday.

A new bank has decided to stay open only on weekends all day Saturday and Sunday, and no other days. The bank has hired two mangers (U and V) for tellers (W,X,Y, and Z), and two operations officers (S and T), for a total of exactly eight full-time employees. No part time employees are hired. Each employee works a complete day when working. A manager must be on duty each day. The mangers cannot work on the same day. At least two tellers must be working on the same day. W and X will not work on the same day. S and Z will only work on Saturday. No employee can work on consecutive days, but each employee must work on Saturday or Sunday. Question:

If W works on Sunday, then which of the following must be true? I. X works on Saturday. II. Y works on Saturday. III. T works on Sunday.
A. I B. II C. III D. I & II E. I & III

Given Answer: Explanation:

Correct Answer

Another possible display might look like this:

Since W and X will not work on the same day, (I) X works on Saturday must be true. (II) is false since Y must work on Sunday. (III) T works on Sunday could be true.

A new bank has decided to stay open only on weekends all day Saturday and Sunday, and no other days. The bank has hired two mangers (U and V) for tellers (W,X,Y, and Z), and two operations officers (S and T), for a total of exactly eight full-time employees. No part time employees are hired. Each employee works a complete day when working. A manager must be on duty each day. The mangers cannot work on the same day. At least two tellers must be working on the same day. W and X will not work on the same day. S and Z will only work on Saturday. No employee can work on consecutive days, but each employee must work on Saturday or Sunday. Question:

Which of the following must be true? I. If U works on Saturday, then V works on Sunday. II. If U works on Sunday, then X works on Saturday. III. If X works on Saturday , then W works on Sunday.
A. I B. II C. III D. I & II E. I And III

Given Answer: Explanation:

Correct Answer

Another possible display might look like this:

From the original information, a manger must be on duty each day and the mangers cannot work on the same day. Therefore, (I) if U works on Saturday, then V works on Sunday, must be true. (II) does not have to be true, since U schedule has no bearing on Xs schedule. Since W and X will not work on the same day, then (III) if X works on Saturday, then W works on Sunday) must also be true.

A new bank has decided to stay open only on weekends all day Saturday and Sunday, and no other days. The bank has hired two mangers (U and V) for tellers (W,X,Y, and Z), and two operations officers (S and T), for a total of exactly eight full-time employees. No part time employees are hired. Each employee works a complete day when working. A manager must be on duty each day. The mangers cannot work on the same day. At least two tellers must be working on the same day. W and X will not work on the same day. S and Z will only work on Saturday. No employee can work on consecutive days, but each employee must work on Saturday or Sunday. Question:

Which of the following is an acceptable group of employees that could work on Saturday?
A. ZWYST B. UVWYZS C. VWXZT D. UZST E. VWZS

Given Answer: Explanation:

Correct Answer

Another possible display might look like this:

V, W, Z,S can work on Saturday without breaking any of the conditions given. Choice (A) is missing a manger. Choice B has two mangers working on the same day. Choices c and D have W and X working on the same day.

A new bank has decided to stay open only on weekends all day Saturday and Sunday, and no other days. The bank has hired two mangers (U and V) for tellers (W,X,Y, and Z), and two operations officers (S and T), for a total of exactly eight full-time employees. No part time employees are hired. Each employee works a complete day when working. A manager must be on duty each day. The mangers cannot work on the same day. At least two tellers must be working on the same day. W and X will not work on the same day.

S and Z will only work on Saturday. No employee can work on consecutive days, but each employee must work on Saturday or Sunday. Question:

Which of the following must be true?


A. T Always Works The Same Day As Y. B. S Never Works The Same Day As U. C. Z Never Works The Same Day As X. D. If W Works On Sunday, Then Y Always Works On Saturday. E. Only Two Tellers Works On Saturday.

Given Answer: Explanation:

Correct Answer

Another possible display might look like this:

Since no employee can work on consecutive days, and there are four tellers, then two must work on Saturday.

A new bank has decided to stay open only on weekends all day Saturday and Sunday, and no other days. The bank has hired two mangers (U and V) for tellers (W,X,Y, and Z), and two operations officers (S and T), for a total of exactly eight full-time employees. No part time employees are hired. Each employee works a complete day when working. A manager must be on duty each day. The mangers cannot work on the same day. At least two tellers must be working on the same day. W and X will not work on the same day. S and Z will only work on Saturday. No employee can work on consecutive days, but each employee must work on Saturday or Sunday. Question:

Which of the following is a complete and accurate list of the employees who have the possibility of working on Sunday?
A. UWYZ B. UWYS C. UVWXT D. UVWXYT E. UVWXYTS

Given Answer: Explanation:

Correct Answer

Another possible display might look like this:

U, V, W, X, Y ,Z and T have the possibility of working on Sunday, S and Z do not.

Kelly, Clyde, Roland, Fred, and Harriet are a lawyer, an accountant, a doctor, a police officer, and a cab deriver, not necessarily in that order. 1. 2. 3. 4. 5. 6. Kelly is not the doctor or the accountant. If Fred is the accountant, then Kelly is the doctor. Roland is not the police officer. If Clyde is the doctor, then Harriet is the lawyer. If Harriet is the cab driver, then Kelly is not the police officer. Harriet is the cab driver.

Question:

If the accountant, the doctor, and the police officer meet for lunch which three people get together?
A. Kelly , Fred, And Clyde B. Roland, Fred And Kelly C. Roland, Clyde And Kelly D. Roland, Fred, And Clyde E. None Of These

Given Answer: Explanation:

Correct Answer

From our chart we can see that the only persons left to be the accountant, the doctor, and the police officer are Clyde, Roland, and Fred.

Kelly, Clyde, Roland, Fred, and Harriet are a lawyer, an accountant, a doctor, a police officer, and a cab deriver, not necessarily in that order. 1. 2. 3. 4. 5. Kelly is not he doctor or the accountant. It Fred is the accountant, then Kelly is the doctor. Roland is not the police officer. If Clyde is the doctor, then Harriet is the lawyer. If Harriet is the cab driver, then Kelly is not the police officer.

6. Harriet is the cab driver. Question:

If Clyde is NOT the accountant, then I. Roland is the accountant II. Clyde is the police officer III. Fred is the doctor.
A. I Only B. II Only C. III Only D. I And III E. I, II And III

Given Answer: Explanation:

Correct Answer

If Clyde is not the accountant, he must be the police officer. Therefore Fred must be the doctor, leaving Roland to be the accountant. Thus of all, I, II, and III are true.

Kelly, Clyde, Roland, Fred, and Harriet are a lawyer, an accountant, a doctor, a police officer, and a cab deriver, not necessarily in that order. 1. 2. 3. 4. 5. 6. Kelly is not he doctor or the accountant. It Fred is the accountant, then Kelly is the doctor. Roland is not the police officer. If Clyde is the doctor, then Harriet is the lawyer. If Harriet is the cab driver, then Kelly is not the police officer. Harriet is the cab driver.

Question:

The doctor could I. Clyde

II. Roland III. Fred


A. I Only B. II Only C. III Only D. I And III E. II And III

Given Answer: Explanation:

Correct Answer

The doctor could be either Roland or Fred.

Kelly, Clyde, Roland, Fred, and Harriet are a lawyer, an accountant, a doctor, a police officer, and a cab deriver, not necessarily in that order. 1. 2. 3. 4. 5. 6. Kelly is not he doctor or the accountant. It Fred is the accountant, then Kelly is the doctor. Roland is not the police officer. If Clyde is the doctor, then Harriet is the lawyer. If Harriet is the cab driver, then Kelly is not the police officer. Harriet is the cab driver.

Question:

Clyde cannot be the I. Accountant II. Police officer III. Doctor


A. I Only B. II Only C. III Only D. I And III E. II And III

Given Answer: Explanation:

Correct Answer

From the chart we can see the Clyde is not the lawyer, the doctor or the cab driver.

Kelly, Clyde, Roland, Fred, and Harriet are a lawyer, an accountant, a doctor, a police officer, and a cab deriver, not necessarily in that order. 1. 2. 3. 4. 5. 6. Kelly is not he doctor or the accountant. It Fred is the accountant, then Kelly is the doctor. Roland is not the police officer. If Clyde is the doctor, then Harriet is the lawyer. If Harriet is the cab driver, then Kelly is not the police officer. Harriet is the cab driver.

Question:

If Roland is the doctor, the Clyde is the


A. Police Officer B. Accountant C. Lawyer D. None Of These E. Not Enough Information To Tell

Given Answer: Explanation:

Correct Answer

If Roland is the doctor, then from our chart we can see that Fred must be the police officer, and thus Clyde must be the accountant.

Kelly, Clyde, Roland, Fred, and Harriet are a lawyer, an accountant, a doctor, a police officer, and a cab deriver, not necessarily in that order. 1. 2. 3. 4. 5. 6. Kelly is not he doctor or the accountant. It Fred is the accountant, then Kelly is the doctor. Roland is not the police officer. If Clyde is the doctor, then Harriet is the lawyer. If Harriet is the cab driver, then Kelly is not the police officer. Harriet is the cab driver.

Question:

If Fred is the police officer, then


A. Roland Could Be The Doctor B. Roland Must Be The Accountant C. Clyde Must Be The Accountant D. Clyde Could Be The Lawyer E. Clyde Must Be The Doctor

Given Answer: Explanation:

Correct Answer

If Fred is the police officer, then Clyde must be the accountant.

Kelly, Clyde, Roland, Fred, and Harriet are a lawyer, an accountant, a doctor, a police officer, and a cab driver, not necessarily in that order. 1. 2. 3. 4. 5. 6. Kelly is not he doctor or the accountant. It Fred is the accountant, then Kelly is the doctor. Roland is not the police officer. If Clyde is the doctor, then Harriet is the lawyer. If Harriet is the cab driver, then Kelly is not the police officer. Harriet is the cab driver.

Question:

If the cab driver picks up the lawyer and the accountant, which three people could be in the cab? I. Harriet II. Fred III. Roland IV. Kelly
A. I, II And III B. II, III, And IV C. I, II, And IV D. I, III, And IV E. None Of These

Given Answer: Explanation:

Correct Answer

If the cab driver, the lawyer, and the accountant are in the cab, then Harriet (cab driver) and Kelly (lawyer) are definitely in the cab. Since the accountant is either Clyde or Roland, then either of them could possibly be also in the cab. Thus , the answer is Harriet, Roland and Kelly(I, III and IV).

Eleven people stand single file in a straight line. There are our women, three men, two boys and two girls. The children are all nest to each other Three of the women stand next to each other at one end of the line. No man stands next to another man. Question:

If a child is in the sixth place, which of the following MUST be true?


A. A Woman Is In The First Place B. A Woman Is In The Second Place C. A Woman Is In The Seventh Place D. A Woman Is In The Eight Place E. A Woman Is In The Ninth Place.

Given Answer: Explanation:

Correct Answer

First, since three of the women are together at one end of the line, these women could be at either the beginning or the end.

Then note that, since a man never stands next to another man, there can only be two possible arrangements of men for each of the above arrangements of women:

Note that if a child is in the sixth place (the middle four possible arrangements) then a woman must be in the second place.

Eleven people stand single file in a straight line. There are our women, three men, two boys and two girls. The children are all nest to each other Three of the women stand next to each other at one end of the line. No man stands next to another man. Question:

Which of the following MUST be true?


A. A Woman Is Either First Or Third B. A Woman Is Either Second Or Tenth C. A Woman Is Either Sixth Or Seventh D. A Woman Is Fourth E. A Woman Is Seventh.

Given Answer: Explanation:

Correct Answer

First, since three of the women are together at one end of the line, these women could be at either the beginning or the end.

Then note that, since a man never stands next to another man, there can only be two possible arrangements of men for each of the above arrangements of women:

Since the rules state the three of the women stand next to each other at one end of the line, then a woman must be either second or tenth (second to last).

Eleven people stand single file in a straight line. There are our women, three men, two boys and two girls. The children are all nest to each other Three of the women stand next to each other at one end of the line. No man stands next to another man. Question:

All of the following MUST be true except


A. If A Man Is First , A Women Is Last B. If A Child Is Eighth, A Man Is Ninth C. If A Child Is Sixth , A Woman Is Tenth D. If A Man Is Fourth, A Child Is Eight E. If A Woman Is Third, A Man Is Sixth

Given Answer: Explanation:

Correct Answer

First, since three of the women are together at one end of the line, these women could be at either the beginning or the end.

Then note that, since a man never stands next to another man, there can only be two possible arrangements of men for each of the above arrangements of women:

All of the choices must be true except choice ( E ). If a woman is third, child could be sixth, as in the second arrangement.

1. 2. 3. 4. 5. 6.

Bill is taller than Ann. Drew is shorter than Bill. Clem is taller than Drew, but shorter than Ann. Ellen is taller than Clem, but shorter than Bill. Fran is taller than Ellen. Gina is shorter than Ann.

Question:

If Ann is 5 feet tall, then which of the following could not be true? I. Fran is 4 feet 10 inches tall. II. Ellen is 5 feet 10 inches tall. III. Drew is 5 feet inches tall.
A. I Only B. II Only C. III Only D. Two Of The Above E. None Of The Above

Given Answer: Explanation:

Correct Answer

We see that Drew is shorter than Ann. Thus II is false. Statements I and II could be true.

1. 2. 3. 4. 5. 6.

Bill is taller than Ann. Drew is shorter than Bill. Clem is taller than Drew, but shorter than Ann. Ellen is taller than Clem, but sheerer than Bill. Fran is taller than Ellen. Gina is shorter than Ann.

Question:

Which of the following could not be true?


A. Drew Is Shorter Than Ann. B. Ellen Is 4 Feet Tall. C. Bill Is Taller Than Fran. D. Fran Is Shorter Than Clem. E. Drew Is 4 Feet Tall.

Given Answer: Explanation:

Correct Answer

Since Fran is taller than Ellen and Ellen is taller than Clem, Fran is taller than Clem.

1. 2. 3. 4. 5. 6.

Bill is taller than Ann. Drew is shorter than Bill. Clem is taller than Drew, but shorter than Ann. Ellen is taller than Clem, but shorter than Bill. Fran is taller than Ellen. Gina is shorter than Ann.

Question:

If Ellen is taller than Ann, then which of the following MUST be true about Fran? I. Fran is taller than Ann. II. Fran is taller than Bill. III. Fran s taller than Drew.
A. I And II B. I And III C. II And II D. II Only E. III Only

Given Answer: Explanation:

Correct Answer

Since Fran is taller than Ellen, and if Ellen is taller than Ann, then Fran must be taller than Ann. Thus I is true. Since Ann is taller than Drew, Fran must be taller than Drew. Thus, III is true. Statement II could be true, but does not have to be.

1. 2. 3. 4. 5. 6.

Bill is taller than Ann. Drew is shorter than Bill. Clem is taller than Drew, but shorter than Ann. Ellen is taller than Clem, but sheerer than Bill. Fran is taller than Ellen. Gina is shorter than Ann.

Question:

Which statement is redundant and repeats information obtainable of the other statements?
A. 1 B. 2 C. 3 D. 4 E. 5

Given Answer: Explanation:

Correct Answer

From statement 3 Drew is shorter than Ann. From statement 1 Ann is shorter than Bill. It follows that Drew is shorter than Bill.

Logical Reasoning
Both Superfit and Fun-Fit have printed a toll-free number on their exercise products to provide roundthe-clock telephone assistance to any customer who encounters difficulties. Since customers only call the numbers when they find the exercise equipment difficult to use, and the Superfit line receives three times as many calls as the Express line, Superfits product must be more difficult to use than Fun-fits. Question:

Which of the following, if true, most strengthens the argument above?


A. Calls To The Fun-Fit Number Are Almost Twice As Long, On Average, Than Calls To The B. Fun-Fit Has Sold Four Times The Number Of Its Exercise Product That Superfit Has. C. Fun-Fit Receives Twice As Many Letters Of Complaint About Its Product As Superfit Receives About Its Product. D. The Number Of Calls Received By Each Of The Two Lines Has Been Gradually Increasing. E. The Superfit Number Is More Widely Publicized That The Fun-Fit Number.

Superfit Line.

Given Answer: Explanation:

Correct Answer

The passage argues that since Superfit receives more telephone calls, the product must be more difficult to use. If Fun-Fit has sold four times the number products as Superfit has sold, this claim is strengthened.

Certain upgrades need to be made on each of the computers owned by Bosco Insurance. The work of upgrading their 20 computers is mentally taxing. Employees are using the computers during the day, making it difficult for the computer technician to coordinate the block of time needed to work on each computer. Therefore, the most efficient time to upgrade the computers is between ten oclock p.m. and eight oclock a.m. when the computers are free. Question:

Which of the following, if true, argues most strongly against the view that upgrades could be done more efficiently by scheduling them at night?
A. Energy Costs In Businesses Are Greatly Lower At Night Than They Are During The Day. B. More Blackouts Occur Between Midnight And Eight Oclock In The Morning T han At Any Other Time. C. Over The Course Of A Year, The Companys Computers Will Require Periodical Checks And Upkeep D. Computer Technicians Are Generally Paid The Same When They Work During The Night Than When They Work During The Day E. Mental Alertness Is Lower In The Late Night Than During The Day, Especially In People

Not Accustomed To Working At Night.

Given Answer: Explanation:

Correct Answer

The conclusion is that night is the most efficient time for the computer technician to work. However, if the technicians mental alertness will be lower, this cast serious doubt that this task could be done most efficiently at night.

Reporter: Argent Antiques has misled its clients by promoting some Chinese porcelain figurines as rare when in fact those pieces are relatively commonplace. Argent dealer: That is absurd! Argent Antiques is the largest antique dealer in the country. We authenticate the figurines we sell through a nationally recognized firm. Question:

The Argent dealer s reply is most vulnerable to the criticism that it


A. Exaggerates The Reporters Claims In Order To Make Them Appear Absurd B. Accuses The Reporter Of Bias But Presents No Evidence To Support That Accusation C. Fails To Establish That Other Antique Dealers Do Not Also Authenticate The Figurines Those Dealers Sell D. Lists Strengths Of Argent Antiques While Failing To Address The Reporters Charge E. Provides No Definition For The Inherently Vague Phrase Rare

Given Answer: Explanation:

Correct Answer

Argent antiques has been accused of promoting some pieces as rare when they are, in fact, common. The dealers reply does not directly address this accusation, rather it only lists the companys strengths.

The cost of manufacturing sports shoes in Macao is 25% less than the cost of manufacturing them in the Philippines. Even after transportation fees and tariff charges are added, it is still cheaper for a company to import sport shoes from Macao to the Philippines than to manufacture sports shoes in the Philippines. Question:

The statements above, if true, best support which of the following assertions?
A. Labor Costs In Macao Are 25% Below Those In The Philippines. B. Importing Sports Shoes From Macao To The Philippines Will Eliminate 25% Of The Manufacturing Jobs In The Philippines. C. The Tariff On Sports Shoes Imported From Macao To The Philippines Is Less Than 25%

Of The Cost Of Manufacturing Sports Shoes In The Philippines. D. The Fee For Transporting A Pair Of Sports Shoes From Macao To The Philippines Is More Than 25% Of Cost Of Manufacturing The Shoes In Macao. E. It Takes 25% Less Time To Manufacture A Pair Of Sports Shoes In Macao Than It Does In The Philippines./Li>

Given Answer: Explanation:

Correct Answer

If the tariff on importing sport shoes from Macao to the Philippines were as high as 25% or more of the cost of producing sports shoes in the Philippines, then, contrary to what the passage says, the cost of importing sports shoes would be equal to or more than the cost of producing sports shoes in the Philippines. Thus, the tariff cannot be that high.

Opponents of laws that require motorcycle riders to wear helmets argue that in a free society people have the right to take risks as long as the people do not harm others as a result of taking the risks. As a result, they conclude that it should be each persons decision whether or not to wear a helmet. Question:

Which of the following, if true, seriously weakens the conclusion drawn above?
A. Many New Motorcycles Are Built With Safety Features That Made Them Less Likely To B. Motorcycle Insurance Rates For All Motorcycle Owners Are Higher Because Of The N eed To Pay For The Increased Injuries Or Deaths Of People Not Wearing Helmets. C. Participants In Certain Extreme Sports Are Required To Wear Helmets. D. The Rate Of Automobile Fatalities In States That Do Not Have Mandatory Helmet Laws Is Greater Than The Rate Of Fatalities In States That Do Have Such Laws. E. In Motorcycle Accidents, A Greater Number Of Passengers Who Do Not Wear Helmets Are Injured Than Are Passengers Who Do Wear Helmets.

Be Involved In An Accident.

Given Answer: Explanation:

Correct Answer

The principle that people are entitled to risk injury provided they do not thereby harm others fails to justify the individuals right to decide not to wear a helmet if it can be shown that it does harm others by raising insurance rates.

A drug that is highly effective in treating certain types of cancerous growth can, at present, be obtained only from the cartilage of a particular sub-species of shark, a sub-species which is quite rare in the wild. One must kill 50 sharks to make one pound of the drug. If follows, therefore, that continued production of the drug must inevitably lead to the extinction of this sub-species of shark.

Question:

Which of the following, if true, most seriously weakens the argument above?
A. The Drug Made From The Shark Cartilage Is Dispensed To Doctors From A Central B. The Drug Made From The Shark Cartilage Is Expensive To Produce. C. Other Organs Of The Shark Can Be Utilized To Produce Different Drugs. D. The Sub-Species Of Shark Will Reproduce In Captivity Under The Proper Conditions. E. This Sub-Species Of Shark Generally Lives In Largely Inaccessible Waters.

Authority.

Given Answer: Explanation:

Correct Answer

If the shark can be successfully bred in captivity, it is possible to continue production of the drug without threatening the shark with extinction.

Harpers Pencils manufactures and sells the same pencils as Johnsons Supply. Employee wages account for forty percent of the cost of manufacturing pencils at both factories. Harpers is seeking a competitive edge over Johnsons supply. Therefore, to promote this end, Harpers should lower employee wages. Question:

Which of the following, if true, seriously weakens the argument above?


A. Because They Make A Small Number Of Specialty Artists Pencils, Pencil Manufacturers B. Lowering Wages Would Reduce The Quality Of Employee Work And This Reduced Quality Would Lead To Lowered Sales. C. Harpers Pencils Has Taken Away Twenty Percent Of Johnsons Supply Business Over The Last Year. D. Johnsons Supply Pays Its Employees, On Average, Ten Percent More Than Does Harpers Pencils. E. Many People Who Work For Manufacturing Plants Live In Areas In Which The Manufacturing Plant They Work For Is The Only Industry.

Cannot Receive Volume Discounts On Raw Materials.

Given Answer: Explanation:

Correct Answer

The effect of lowering wages is to reduce quality sufficiently to reduce sales. This is a good reason to doubt that wage cuts would give Johnson any competitive edge.

The petrochemical industry claims that chemical waste dumps pose no threat to people living near

them. If this is true, then why do they locate the plants in sparsely populated regions? By not locating the chemical dumps in densely populated areas the petrochemical industry tacitly admits that these chemicals are potentially dangerous to the people living nearby. Question:

Which of the following, if true, would most weaken the author's argument?
A. Funding Through The Environmental Super Fund To Clean Up Poorly Run Waste Dumps B. Until Chemical Dumps Are Proven 100% Safe, It Would Be Imprudent To Locate Them Were They Could Potentially Do The Most Harm. C. Locating The Dumps In Sparsely Populated Areas Is Less Expensive And Involves Less Government Red Tape. D. The Potential For Chemicals To Leach Into The Water Table Has In The Past Been Underestimated. E. People In Cities Are More Likely To Sue The Industry If Their Health Is Harmed By The Dumps.

Is Reserved For Rural Areas Only.

Given Answer: Explanation:

Correct Answer

The suppressed false premise of the argument is that all things being equal there is no reason to prefer locating the sites in sparsely populated areas. To weaken the argument, we need to show it is not true that all things are equal. In other words, there are advantages other than safety in locating the sites in sparsely populated areas. Choice (C) gives two possible advantagescost and ease. Hence (C) is the answer.

Plants that exhibit certain leaf diseases tend to measure extremely high in the amount of zinc in their leaf and stem tissue. Botanists have discovered that phosphorus of the type typically used in a phosphorus-high fertilizer reacts with the zinc in such a way as to prevent treated plants from exhibiting the leaf diseases. Thus, plants can be cured from these leaf diseases by the use of a fertilizer high in phosphorus. Question:

The passages conclusion is based upon which one of the following premises?
A. Plants With Certain Leaf Diseases Contain The Same High Level Of Zinc In Their Leaf B. Zinc Is The Cause And Not Merely An Effect Of The Leaf Diseases. C. Treating The Plants With A Fertilizer High In Phosphorus Will Have No Negative Effect On The Plants. D. The Amount Of Phosphorus-High Fertilizer Which Should Be Used Depends Upon The Size And Location Of The Plants. E. Normal Plant Tissue Does Not Contain Zinc.

And Stem Tissue.

Given Answer: Explanation:

Correct Answer

In this question, the premise has been suppressed. If the high level of zinc found in diseased plants is merely a symptom of the diseases, and zinc does not actually cause the diseases, then the fact that a fertilizer high in phosphorus causes plants not to exhibit the diseases, is irrelevant. So the question is based upon the assumption set forth in choice (B). The answer is (B).

The petrochemical industry claims that chemical waste dumps pose no threat to people living near them. If this is true, then why do they locate the plants in sparsely populated regions? By not locating the chemical dumps in densely populated areas the petrochemical industry tacitly admits that these chemicals are potentially dangerous to the people living nearby. Question:

Which of the following, if true, would most weaken the author's argument?
A. Funding Through The Environmental Super Fund To Clean Up Poorly Run Waste Dumps B. Until Chemical Dumps Are Proven 100% Safe, It Would Be Imprudent To Locate Them Were They Could Potentially Do The Most Harm. C. Locating The Dumps In Sparsely Populated Areas Is Less Expensive And Involves Less Government Red Tape. D. The Potential For Chemicals To Leach Into The Water Table Has In The Past Been Underestimated. E. People In Cities Are More Likely To Sue The Industry If Their Health Is Harmed By The Dumps.

Is Reserved For Rural Areas Only.

Given Answer: Explanation:

Correct Answer

The suppressed false premise of the argument is that all things being equal there is no reason to prefer locating the sites in sparsely populated areas. To weaken the argument, we need to show it is not true that all things are equal. In other words, there are advantages other than safety in locating the sites in sparsely populated areas. Choice (C) gives two possible advantagescost and ease. Hence (C) is the answer.

You might also like